Angle of minimum deviation for a prism refractive index \(1.5\) is equal to the angle of the prism. Then the angle of prism (given, $\sin 48^{\circ} 36^{\prime}=0.75$ ) (A) \(62^{\circ}\) (B) \(82^{\circ}\) (C) \(60^{\circ}\) (D) \(41^{\circ}\)

Short Answer

Expert verified
The angle of the prism is \(60^{\circ}\), which corresponds to answer choice (C).

Step by step solution

01

Write down the formula for angle of minimum deviation in a prism

For a prism with refractive index n, angle of prism A, and angle of minimum deviation \(\delta\), the formula is: \(n = \frac{\sin(A + \delta) / 2}{\sin{A / 2}}\) Here, we are given \(n = 1.5\) and \(\delta = A\).
02

Substitute the given values into the formula

Substitute the given values into the formula: \(1.5 = \frac{\sin(A + A) / 2}{\sin{A / 2}}\) Since \(\delta = A\), we have: \(1.5 = \frac{\sin(2A) / 2}{\sin{A / 2}}\)
03

Eliminate the denominators

Now, we can eliminate the denominators by multiplying both sides by \(2\sin(A / 2)\): \(3\sin(A / 2) = \sin(2A)\)
04

Write the sine of double angle in terms of sin and cos of the angle

Recall the double-angle formula for sine: \(\sin(2A) = 2\sin(A)\cos(A)\) Now substitute this back into our equation: \(3\sin(A / 2) = 2\sin(A)\cos(A)\)
05

Use given sine of angle of minimum deviation

We know that for the angle of minimum deviation, its sine is: \(\sin(48^\circ 36') = 0.75\) Since the angle of minimum deviation is equal to the prism angle, A, we have: \(\sin(A) = 0.75\)
06

Solve for angle A

Now, we need to rewrite our equation from step 4 using the given sine of angle A and solve for angle A: \(3\sin(A/2)=2(0.75)\cos(A)\) \(3\sin(A/2)=1.5\cos(A)\) Since we know that \(\sin^2(x) + \cos^2(x) = 1\), we can find the cosine of angle A: \(\cos(A)=\sqrt{1-\sin^2(A)}=\sqrt{1-0.75^2}\) Now substitute the cosine of angle A into the equation: \(3\sin(A/2)=1.5\sqrt{1-0.75^2}\) Solving for \(\sin(A/2)\), we find: \(\sin(A/2)=\frac{1.5\sqrt{1-0.75^2}}{3}=\frac{1}{2}\) Finally, we can find the angle A: \(A=2\arcsin(\frac{1}{2})\) \(A=2 * 30^{\circ}\) \(A=60^{\circ}\) So, the angle of the prism is \(\boxed{60^{\circ}}\), which corresponds to answer choice (C).

Unlock Step-by-Step Solutions & Ace Your Exams!

  • Full Textbook Solutions

    Get detailed explanations and key concepts

  • Unlimited Al creation

    Al flashcards, explanations, exams and more...

  • Ads-free access

    To over 500 millions flashcards

  • Money-back guarantee

    We refund you if you fail your exam.

Over 30 million students worldwide already upgrade their learning with Vaia!

One App. One Place for Learning.

All the tools & learning materials you need for study success - in one app.

Get started for free

Most popular questions from this chapter

A mark at the bottom of the liquid appears to rise by \(0.2 \mathrm{~m}\), If depth of the liquid is \(2.0 \mathrm{~m}\) then refractive index of the liquid is (A) \(1.80\) (B) \(1.60\) (C) \(1.33\) (D) \(1.11\)

A light of wavelength \(320 \mathrm{~nm}\) enters in a medium of refractive index \(1.6\) from the air of refractive index \(1.0\). The new wavelength of light in the medium will be \(\mathrm{nm}\). (A) 520 (B) 400 (C) 320 (D) 200

$$ \begin{array}{|l|l|} \hline \text { Column - I } & \text { Column - II } \\ \hline \text { (i) While going from rarer to denser medium } & \text { (a) Wavelength changes } \\ \text { (ii) While going from denser to rarer medium } & \text { (b) } \eta=(\mathrm{C} / \mathrm{V}) \\ \text { (iii) While going to one medium to another } & \text { (C) Ray bends towards normal } \\ \text { (iv) Refractive index of medium } & \text { (D) Rav bends awav from normal } \\ \hline \end{array} $$ (A) \(i-c\), ii \(-d\), iii \(-b\), iv-a (B) \(\mathrm{i}-\mathrm{a}\), ii \(-\mathrm{b}\), iii $-\mathrm{c}, \mathrm{iv}-\mathrm{d}$ (C) $\mathrm{i}-\mathrm{c}, \mathrm{ii}-\mathrm{b}, \mathrm{iii}-\mathrm{a}, \mathrm{iv}-\mathrm{d}$ (D) \(i-d, 1 i-c, 11 i-b, i v-a\)

A convex lens of glass \((\mathrm{n}=1.5)\) has focal length \(0.2 \mathrm{~m}\). The lens is immersed in water of refractive index \(1.33\). The change in the power of convex lens is (A) \(3.72 \mathrm{D}\) (B) \(4.62 \mathrm{D}\) (C) \(6.44 \mathrm{D}\) (D) \(1.86 \mathrm{D}\)

A ray of light is incident at an angle \(30^{\circ}\) on a mirror, The angle between normal and reflected ray is (A) \(15^{\circ}\) (B) \(30^{\circ}\) (C) \(45^{\circ}\) (D) \(60^{\circ}\)

See all solutions

Recommended explanations on English Textbooks

View all explanations

What do you think about this solution?

We value your feedback to improve our textbook solutions.

Study anywhere. Anytime. Across all devices.

Sign-up for free